Вы находитесь на странице: 1из 37

Unit 551

July 2018

Ophthalmology

www.racgp.org.au/check
Disclaimer

The information set out in this publication is current at the date of first publication and is intended
for use as a guide of a general nature only and may or may not be relevant to particular patients
or circumstances. Nor is this publication exhaustive of the subject matter. Persons implementing
any recommendations contained in this publication must exercise their own independent skill or
judgement or seek appropriate professional advice relevant to their own particular circumstances
when so doing. Compliance with any recommendations cannot of itself guarantee discharge of the
duty of care owed to patients and others coming into contact with the health professional and the
premises from which the health professional operates.

Whilst the text is directed to health professionals possessing appropriate qualifications and skills
in ascertaining and discharging their professional (including legal) duties, it is not to be regarded
as clinical advice and, in particular, is no substitute for a full examination and consideration of
medical history in reaching a diagnosis and treatment based on accepted clinical practices.

Accordingly, The Royal Australian College of General Practitioners Ltd (RACGP) and its
employees and agents shall have no liability (including without limitation liability by reason of
negligence) to any users of the information contained in this publication for any loss or damage
(consequential or otherwise), cost or expense incurred or arising by reason of any person using or
relying on the information contained in this publication and whether caused by reason of any error,
negligent act, omission or misrepresentation in the information.

Subscriptions
For subscriptions and enquiries please call 1800 331 626 or email check@racgp.org.au

Published by
The Royal Australian College of General Practitioners Ltd
100 Wellington Parade
East Melbourne, Victoria 3002, Australia

Telephone 03 8699 0414


Facsimile 03 8699 0400
www.racgp.org.au

ABN 34 000 223 807


ISSN 0812-9630

© The Royal Australian College of General Practitioners 2018

This resource is provided under licence by the RACGP. Full terms are available at www.racgp.
org.au/usage/licence. In summary, you must not edit or adapt it or use it for any commercial
purposes. You must acknowledge the RACGP as the owner.

We acknowledge the Traditional Custodians of the lands and seas on which we work and live,
and pay our respects to Elders, past, present and future.
Ophthalmology
Unit 551 July 2018

About this activity 2

Case 1 Luke’s left eye has a white reflection 5

Case 2 Tom’s right eye is painful 10

Case 3 Jack presents with sudden, sharp eye pain 15

Case 4 Meryl has unilateral blurred vision 20

Case 5 Danny has itchy eyes 25

Case 6 Cynthia has type 2 diabetes 28

Multiple choice questions 32

The five domains of general practice

Communication skills and the patient–doctor relationship


Applied professional knowledge and skills
Population health and the context of general practice
Professional and ethical role
Organisational and legal dimensions
WHAT WILL YOUR PATIENTS MISS MOST 
IF YOU MISS THEIR RETINOPATHY?
Patients with type 2 diabetes should undergo regular screening
to diagnose and manage diabetic retinopathy early1,2

Lipidil is indicated for the reduction in the progression of diabetic retinopathy (DR)
in patients with type 2 diabetes and existing DR.3 Patients with mild non-proliferative
diabetic retinopathy received the greatest reduction in DR progression.4
Lipidil does not replace the appropriate control of blood pressure, blood glucose
and blood lipids in reducing the progression of DR.3

PBS Information: Restricted Benefit. For use in patients that meet the criteria set out in the
General Statement for Lipid-Lowering Drugs. Not listed for the treatment of diabetic retinopathy.

Please review the full Product Information (PI) before prescribing. Full PI available on request by calling
1800 314 527 or at: www.medicines.org.au
Lipidil® (fenofibrate) 145mg tablets, 30’s and 48mg tablets, 60’s. INDICATIONS: Adjunct to diet in the treatment of hypercholesterolaemia, types II, III, IV and V dyslipidaemia,
dyslipidaemia associated with type 2 diabetes. Reduction in the progression of diabetic retinopathy in patients with type 2 diabetes. Does not replace appropriate control of blood pressure,
blood glucose and blood lipids. DOSAGE: Dyslipidaemia and Diabetic Retinopathy: One 145 mg tablet once daily with or without food. Start with one 48 mg tablet once daily in patients with
moderate renal impairment (CrCl >30ml/min, ≤ 60ml/min), refer to full PI*. CONTRAINDICATIONS: Children; liver dysfunction; severe renal impairment; existing gallbladder disease; co-
administration with another fibrate; hypersensitivity to fibrates or ketoprofen; chronic or acute pancreatitis with the exception of acute pancreatitis due to severe hypertriglyceridaemia.
PRECAUTIONS: Attempt diet and lifestyle modifications before initiating therapy for dyslipidaemia; effect on CHD mortality/morbidity not established; renal impairment; may increase LFT;
hepatic impairment, cholelithiasis; haematologic changes; paradoxical decreases in HDL-C; pregnancy and lactation; drugs exacerbating hypertriglyceridaemia (oestrogen, β-blocker,
thiazides); fructose and/or galactose intolerance; lecithin or related product allergy. INTERACTIONS: Oral anti-coagulants; HMG-CoA reductase inhibitors (risk of muscle toxicity is
increased if used concurrently); other fibrates; cyclosporin (monitor renal function); phenylbutazone; drugs metabolised by cytochrome P450 isoenzymes CYP2C19, CYP2A6, and
CYP2C9. ADVERSE EFFECTS: GI disorders; skin reactions (including rash, photosensitivity, severe cutaneous reactions); raised LFT; increase in serum creatinine; pancreatitis; gallstones;
thromboembolism; muscle toxicity; increase in serum homocysteine and rarely rhabdomyolysis. Min PI Updated: 10 Jan 2018.
*Please note changes in PI
References: 1. The Royal Australian College of General Practitioners. General practice management of type 2 diabetes: 2016–18. East Melbourne, Vic: RACGP, 2016. 2. Fong DS et al.
Diabetes Care 2004;27:S84-S87 3. Lipidil Approved Product Information 4. Chew EY et al. Ophthalmology 2014;121:2443-51.
LIPIDIL® is a registered trademark. Mylan Health Pty Limited. ABN 29 601 608 771.
Level 1, 30 The Bond, 30-34 Hickson Road, Millers Point NSW 2000, Australia Tollfree: 1800 274 276.
Date prepared: April 2018. MYL0051_AFP_FP_IFC. LIP-2018-0006.
Ophthalmology check About this activity

About this activity 2. National Diabetes Education Program diseases of the retina, including
(NDEP). Diabetes and you: Health eyes macular degeneration, retinal vein
In Australia, 2.2% of general practice matter! Georgia: National Center for
Chronic Disease Prevention and Health
occlusion and diabetic retinopathy. He
consultations are related to eye health.1
Promotion, Division of Diabetes has considerable expertise in
Many eye conditions require referral to
Translation, 2014. Available at https:// administering intravitreal injections to
an optometrist or ophthalmologist. www.cdc.gov/diabetes/ndep/pdfs/ treat these conditions. Dr Friedrich
toolkits/working-together/149-healthy-
With the increase in the incidence of commonly receives referrals of tertiary
eyes-matter.pdf [Accessed 07 June 2018].
type 2 diabetes there has been an cases for YAG laser capsulotomy
3. The Royal Australian College of General
increase in the incidence of diabetes- following cataract surgery. Dr Friedrich
Practitioners. General practice
related eye conditions such as diabetic management of type 2 diabetes. is a visiting medical officer at Mater
retinopathy.2 Regular eye checks for Melbourne: RACGP, 2016. Available at Misericordiae Hospital in Sydney and
patients with diabetes are crucial to https://www.racgp.org.au/your- practises at the Vision Eye Institute
practice/guidelines/diabetes [Accessed Chatswood and Mosman clinics.
ensure that visual abnormalities are 7 June 2018].
diagnosed at early stages to optimise Nabill Jacob (Case 6) DOBA, BAppSc
4. Karaconji T, Maconochie Z, McCluskey P.
patient education and treatment.3 Acute anterior uveitis in Sydney. Ocul (Orthoptics), MCommHlth, MIP
Diabetes-related eye conditions Immunol Inflamm 2013;21(2):108–114. doi (ACHSM), is the clinical relationship
accounted for 9.9% of ophthalmologist :10.3109/09273948.2012.745882.
manager at Vision Australia. Mr Jacob
referrals in 2013–14.1 5. Balmer A, Munier F. Differential
has held the posts of senior orthoptist
diagnosis of leukocoria and strabismus,
A painful red eye may be due to an easily first presenting signs of
at St Vincent’s Hospital Sydney, and
treatable condition such as a retinoblastoma. Clin Ophthalmol corneal transplant coordinator at the
conjunctivitis infection; conversely, it 2007;1(4):431–9. Save Sight Institute at Sydney Eye
could be due to uveitis, which is the 6. Batur M, Seven E, Esmer O, Akaltun MN, Hospital. His interests lie in the
Yasar T, Cinal A. Epidemiology of adult advancement of services and clinical
most common inflammatory eye
open globe injury. J Craniofac Surg understanding in the area of low
condition and can cause significant 2016;27(7):1636–41.
vision and blindness. Mr Jacob
visual impairment.4 A white pupil reflex 7. Bashour, M. Ophthalmic manifestations
conducts low vison continuing
may indicate a number of conditions of chlamydia. Medscape, 2016. Available
at https://reference.medscape.com/ professional development sessions for
including Coat’s disease, persistent
article/1203385-overview [Accessed 7 ophthalmic registrars, general
hyperplastic primary vitreous,
June 2018]. practitioners, orthoptists, optometrists
toxoplasmosis, retinal disease or
8. Bhosai SJ, Bailey RL, Gaynor BD, and allied health professionals. Mr
retinoblastoma; urgent referral to a
Lietman TM. Trachoma: An update on Jacob is currently on the steering
specialist is needed.5 prevention, diagnosis and
committee of the NSW Health
treatment. Curr Opin Ophthalmol
Penetrating eye injury should be 2012;23(4):288–95. doi:10.1097/
Department’s – Agency for Clinical
considered on presentation of sudden, ICU.0b013e32835438fc. Innovation (Ophthalmology Network),
sharp eye pain, particularly in men aged 9. Bond-Taylor M, Jakobsson G, Zetterberg a project to develop a resource for
17–29 years.6 This presentation requires M. Posterior vitreous detachment – medical, nursing and ancillary staff at
urgent specialist attention to minimise prevalence of and risk factors for retinal public hospitals. and to enhance their
tears. Clin Ophthalmol 2017;11:1689–95. understanding of the needs and
long-term effects on vision.
doi:10.2147/OPTH.S143898.
treatment of patients who are blind or
Trachoma can cause scarring, trichiasis
have low vision.
and blindness if untreated.7 However, Learning outcomes
early-stage trachoma can be treated with Lee Lenton (Case 3) MBBS, FRANZCO,
At the end of this activity, participants
antibiotics, and blindness from trachoma FRACS is a highly respected and
will be able to:
is preventable,8 emphasising the need experienced cataract and refractive
for proper diagnosis and management. • discuss the management of surgeon. Since 1995, he has performed
retinoblastomas over 15,000 LASIK laser eye surgery
Posterior vitreous detachment (PVD) is a procedures. Dr Lenton also specialises
common age-related condition, affecting • summarise the presentation of retinal
in refractive lens exchange and cataract
24% of people aged 50–59 years and up detachment
surgery, with expertise in trifocal
to 87% of people aged 80–89 years.9 • discuss the management of diabetic intraocular lens technology. Dr Lenton is
Careful management is required, as retinopathy a visiting clinician and lecturer in the
cases of PVD may be uncomplicated or Department of Optometry and the
lead to retinal complications. • identify risk factors for trachoma
Optometry Clinic at Queensland
• outline the treatment of penetrating University of Technology. Dr Lenton
References eye injuries. practises at Vision Eye Institute
1. Britt H, Miller GC, Henderson J, Brisbane and Townsville clinics.
et al. General practice activity in Authors
Australia 2013–14. General practice
Eric Mayer (Case 1) BMBCh, PhD,
series no. 36. Sydney: Sydney University Jeff Friedrich (Case 4) MBBS, FRCOphth, FRANZCO has many years
Press, 2014.   FRANZCO specialises in medical of experience performing routine and

3
About this activity check Ophthalmology

complex ophthalmic surgery. Dr Mayer been a reviewer for The Royal


specialises in the diagnosis and Australian College of General
treatment of all retinal conditions, Practioners since 2014. Her previous
including macular degeneration, philanthropic work in the area of
diabetic retinopathy, retinal ‘development for universal eyecare’
detachment, retinal vein occlusion and was done with Rotary International and
macular hole. He provides both medical John Flynn Placement Program, and
and surgical treatment of these she continues this work today
conditions, with expertise in supporting organisations including
vitreoretinal surgery, laser treatment Sight for All and the Macular
and intravitreal injections. Dr Mayer is Degeneration Foundation.
also a cataract surgeon and performs
Neil Sharma MBBS (Hons), MPH,
laser cataract surgery. Dr Mayer
MMed (Refract Surg), FRANZCO is an
practises at Vision Eye Institute
ophthalmologist at Eye and Retina
Blackburn South and Box Hill clinics.
Specialists Green Square, Sydney and a
Jackie Mein (Case 5) MBBS, FACShm, consultant ophthalmologist at
FAFPHM, MAE is a public health and Westmead Hospital. His special
sexual health physician. She has interests include general ophthalmology,
worked across much of Northern diabetic retinopathy, macular
Australia and is currently the Director of degeneration and cataract surgery.
Medical Services at Wuchopperen
Health Service, Cairns. She has Abbreviations
particular interests in rural and remote
AAC acute angle-closure
health, women’s health and Aboriginal
CSME clinically significant
and Torres Strait Islander health.
macula oedema
Nima Pakrou (Case 2) MBBCh, MMed, CT computed tomography
FCSOphth, FRCSEd, FRANZCO is an GP general practitioner
experienced ophthalmologist with HbA1c glycated haemoglobin
expertise in a range of eye conditions. MBS Medicare Benefits Schedule
Dr Pakrou’s subspecialist interests MRI magnetic resonance imaging
include medical retinal diseases (in NPDR non-proliferative
particular macular degeneration and diabetic retinopathy
diabetic retinopathy), eyelid/lacrimal PHPV persistent hyperplastic
surgery and intraocular inflammation primary vitreous
(eg uveitis). He also specialises in PDR proliferative diabetic
complex cataract surgery. Dr Pakrou is retinopathy
a lecturer and examiner for the PVD posterior vitreous detachment
Australian College of Optometry and a RoP retinopathy of prematurity
consultant ophthalmologist at the Royal RPE retinal pigment epithelium
Victorian Eye and Ear Hospital, the SIAC selective intra-arterial
Alfred Hospital and Launceston chemotherapy
General Hospital. Dr Pakrou practises T2DM type 2 diabetes mellitus
at Vision Eye Institute Footscray. VEGF vascular endothelial
growth factor
Peer reviewers WHO World Health Organization

Christolyn Raj MBBS (Hons), MMED,


Acknowledgement
MPH, FRANZCO, FAAO is a
Melbourne‑based ophthalmologist. We are grateful for the assistance of
Her subspecialty interests are Vision Eye Institute in providing cases
cataract/refractive surgery and retinal 1–4.
disease including diabetes and age-
related macular degeneration. Dr Raj is
an affiliate of the research group at the
Diabetes Institute of Victoria and an
honorary lecturer at the University of
Melbourne. Dr Raj has authored and
co-authored several papers in
prestigious scientific journals and has

4
Ophthalmology check Case 1

CASE Further information

1
On further questioning, Monique says that she sometimes
Luke’s left eye has thinks Luke’s eye seems to be turning outwards. She has
a white reflection noticed this may be happening more consistently now.

Monique, 30 years of age, brings her two-year-old son


Luke to see you for a check-up. She has noticed that
Question 3
when looking at him at certain angles or under bright
lights, his left pupil seems to have a white reflection. What does the appearance of this new sign suggest for Luke?
Recently, she has noticed that his pupil looks white on
some photographs. He does not appear to be light-
sensitive or have any particular symptoms (Figure 1).

Figure 1. Abnormal white reflex in left eye

Question 1
Question 4
What conditions may cause a white pupil?
How may you test Luke’s vision in the clinic?

Further information

Question 2 On ophthalmoscopy, there appears to be a single large,


yellowish mass pushing away from the left retina.
How may you differentiate these conditions in the general
practice setting?

Question 5
Does this finding provide confirmation of the diagnosis?

5
Case 1 check Ophthalmology

Further information • Coats’ disease (a severe exudative form of retinal


telangiectasia)
Luke is referred to an ophthalmologist for specialist
investigation, which confirms the diagnosis of retinoblastoma. • retinal detachment
• retinopathy of prematurity (RoP)
• coloboma
Question 6
• toxocariasis (a retinal infection caused by roundworms
What are the basic treatment options for Luke?
routinely found in dogs and cats).

Answer 2
The patient’s history may provide helpful direction towards
diagnosis and the urgency of referral. RoP occurs in infants
born prematurely and before their retinal vessels have
completed normal growth. The condition involves a
neovascular response to hypoxia in the retina, which may
eventually result in scarring or retinal detachment. The
condition, therefore, is accompanied by a history of
prematurity (<27 weeks gestation) and low birthweight
(<1200 g). The duration of oxygen supplementation is
considered an additional risk factor; however, the impact is
variable. RoP is often bilateral and actively screened for in
Further information
at-risk individuals.
Luke undergoes selective intra-arterial chemotherapy (SIAC),
Cataract may present as a unilateral or bilateral condition. The
which is successful in treating his tumour.
condition may commonly present as an isolated finding.
However, a congenital cataract may also be associated with
concurrent ocular (more common with unilateral cataract) or
Question 7 systemic disease (bilateral cataract). In hereditary conditions,
there may be a family history.
What is the long-term prognosis for Luke’s health and vision?
What follow-up is required? Retinal detachment may be idiopathic but is most commonly
a result of concurrent disease or trauma. Retinal detachment
is routinely unilateral, although it may be bilateral. In children,
trauma is a major risk factor; therefore, a specific history of a
direct or indirect blow is often readily identified. Other risk
factors include congenital anomalies, such as Marfan’s
syndrome or Stickler’s syndrome.

Retinoblastoma is the most common primary malignant


intraocular tumour of childhood, although it is rare within the
population. Approximately 60% of cases will be unilateral,
with the highest incidence occurring in patients under the
age of four years. The majority of unilateral cases are
sporadic in nature. Bilateral involvement is suggestive of a
hereditary trait, demonstrating the importance of an
understanding of family history.1 Bilateral disease will
generally develop earlier in patients.
CASE 1 Answers
Ocular toxocariasis is a rare infection caused by
roundworms. The condition usually occurs through contact
Answer 1 with infected faecal material, which may occur in play
conditions such as a sandpit. Ownership of or exposure to
The most important condition to exclude in a child with a
animals, particularly young dogs and cats, is therefore a risk
white reflex (leukocoria) is retinoblastoma. A white pupil
factor. Geographic location, through its correlation with
reflex, however, may be caused by several conditions that
contact with animals, may further increase the risk (eg rural
must be considered as potential differential diagnoses.
settings). This condition is almost always unilateral in
These include: presentation. Given the concurrent inflammation, patients
may also complain of pain and light sensitivity. Coats’
• congenital cataract
disease is considered one of the main differential diagnoses
• persistent hyperplastic primary vitreous (PHPV) to retinoblastoma.2,3
Ophthalmology check Case 1

Coats’ disease is characterised by the progressive of the retina. Several diagnoses can lead to retinal detachment.
development of abnormal retinal blood vessels. The condition Retinoblastoma may present as a single mass or multiple nodular
often appears later in childhood and is more prevalent in masses. Retinoblastoma can intrude or ‘seed’ into the vitreous,
males, with a frequency 10 times greater than in females.2 The leading to additional opacities, making examination of the retina
condition is unilateral in the majority of cases. difficult, or haze within the vitreous. Conversely, the tumour may
extend beneath the retina, which may lead to retinal detachment.
Observation represents an important opportunity to
The tumour may also spread locally into the optic nerve or the
differentiate conditions, although this can be difficult in a
orbit and to other sides around the body.
primary care setting, especially in young children. Cataract may
be identified by examination, achieved by dilating the pupil and
Answer 3
viewing the appearance of the ‘red reflex’ through a direct
ophthalmoscope. However, pathology occurring behind the An eye appearing to move outward (intermittent exotropia)
cataract may be more difficult to establish (Figure 2). PHPV is may result from a congenital extraocular muscle defect. In
caused by a failure of regression of the primary vitreous. PHPV Luke’s case, his increasing exotropia may reflect progressive
is commonly unilateral in presentation and typically occurs in a visual impairment. The presence of leucocidin and exotropia is
small eye (microphthalmia), which may lead to noticeable strongly suggestive of significant visual pathology, causing a
asymmetry in the appearance/shape between the eyes on change in visual behaviour, and requires further investigation.
presentation. In some instances of PHPV the pupil may be
Anecdotally, patients with intermittent exotropia due to ocular
irregular and there may be an opacity on the cornea. If vision is
imbalance or weakness commonly close one eye when exposed
poor, an afferent pupil defect may be apparent. Observation
to bright sunlight, although the reason for this is not always
should also rule out the presence of inflammation or proptosis,
clear. Parents or carers often notice this sign.
which may be present with infection.

The appearance of the white reflex may also provide clues for Answer 4
diagnosis. Coats’ disease, for example, may present as a grey-
Visual acuity provides a gauge of unilateral or bilateral vision
to-pink reflex rather than the classic white appearance of
loss. The method of visual testing is determined by the age of
retinoblastoma. The colour of the reflex will be determined by
the patient. In babies, preferential looking and observation
the location (central versus peripheral) and extent of the area
may represent testing options, as may objection to occlusion.
affected by the condition.
In cases of significant unilateral vision loss, covering the good
Ophthalmoscopy will often help to establish a provisional eye may cause the patient distress, compared with little or no
diagnosis, but in all instances where pathology is found in the effect of covering the eye with suspected loss, thereby
retina, referral to an ophthalmologist is required. Congenital providing basic feedback. Monitoring the patient’s ability to
cataract typically presents as a central opacification of the lens. scan and pick up objects of different sizes may provide an
Retinal detachment presents as a billowing layer. On additional clue. Recording should be simple (eg ‘identifies
ophthalmoscopy this area may be out of focus with the remainder 100s and 1000s with right eye’; Figure 3). Preschool children

Figure 2. Congenital cataract through dilated pupil Figure 3. Vision testing using 100s and 1000s as a basic guide to acuity
Case 1 check Ophthalmology

can usually match symbols or letters adequately. The distance Answer 5


at which the test is performed should be noted (Figure 4). For
The finding of a large mass is highly suggestive of a tumour
children, a distance of 3 m is usually adequate. When using
(retinoblastoma; Figure 5). This needs to be excluded in the
symbols, an attempt should be made to determine at what
first instance with urgent referral to an ophthalmologist.
distance the child can identify a shape; as the shapes become
Although retinoblastoma is the most likely diagnosis,
smaller in size, the child may repeatedly answer ‘circle’ – this
referred cases may ultimately be diagnosed as other
is an indication that they are unable to identify key differences.
conditions. Coats’ disease remains one of the most common
simulating conditions.2–4

The use of additional imaging may be required to confirm


diagnosis and the extent of any lesion, including size, location
and staging. Systematic review has suggested little benefit
from computed tomography (CT) scanning over magnetic
resonance imaging (MRI) as a diagnostic tool. The superior
soft tissue resolution with MRI helps to identify the presence
and extent of tumour growth outside the globe, where it
cannot be visualised directly. The screening protocol for
retinoblastoma involves MRI, ultrasonography and
examination under anaesthesia.5

Answer 6
The treatment priority on diagnosis of retinoblastoma is the
preservation of life, the eye and, finally, vision, in that order. In
recent times there has been a shift towards eye-preserving
treatments, but these depend on early identification and
prompt referral.

Current therapies include systemic, SIAC and intravitreal


chemotherapy and radiotherapy. SIAC delivers a targeted
approach to drug delivery to the eye via the ophthalmic
artery, thereby minimising systemic effects of treatment,
compared with a systemic chemotherapeutic approach.
Retinoblastoma with vitreous involvement (seeding)
represents a significant treatment challenge. SIAC often
provides limited penetration and efficacy of the drug in the
vitreous. Intravitreal injection represents an additional
complementary therapeutic option in these cases. Radiation
therapy retains a role in selected patients, particularly those
with recurrent or residual tumours following chemotherapy.6
Figure 4. Using a chart to match symbols in infants or young children Enucleation remains the standard of care for advanced
tumours, including those tumours that have penetrated the
extraocular tissue.

Answer 7
Following remission of Luke’s tumour through SIAC
treatment, his long-term health outlook remains positive, with
outcomes following successful treatment improving markedly
in recent years. Five-year survival rates in developed countries
remain as high as 95%.7 Delay in treatment represents a
significant risk, as do socioeconomic and cultural
considerations, which continue to influence the decision to
undertake treatment, particularly enucleation.

Lifelong follow-up remains essential for all patients previously


treated for retinoblastoma, with increased vigilance required
for patients with hereditary involvement. This will need to be
done in concert with the treating ophthalmologist.
Figure 5. Retinoblastoma Surveillance can be every one to three months following
treatment, then every four to six months and then annually.

8
Ophthalmology check Case 1

This will be titrated to the individual patient.

Hereditary retinoblastoma represents a lifetime risk of


developing secondary and new primary malignancies,
including osteosarcoma, primitive neuroectodermal tumours
and melanomas.8 Visual outcomes depend on the size,
location and extension of the original tumour. In mild-to-
moderate retinoblastoma, up to 53% of patients may retain
visual acuities of 6/12 or better, which may be compatible
with the legal driving limit in Australia.9 In more significant
disease or those cases with macular involvement, obtaining
vision better than 6/60 remains unlikely and >50% of eyes
remain below this mark.10,11

Resources for patients


• Cancer Australia, https://childrenscancer.canceraustralia.
gov.au/types-childrens-cancers/retinoblastoma

Resources for doctors


• American Academy of Ophthalmology, http://eyewiki.org/
Retinoblastoma

References
1. Abramson DH, Schefler AC. Update on retinoblastoma. Retina
2004;24:828–48.
2. Shields JA, Shields CL, Honavar SG, Demirci H. Clinical variations
and complications of Coats disease in 150 cases: The 2000
Sanford Gifford Memorial Lecture. Am J Ophthalmol
2001;131(5):561–71.
3. Maki JL, Marr BP, Abramson DH. Diagnosis of retinoblastoma:
how good are referring physicians? Ophthalmic Genet
2009;30(4):199–205.
4. Ghassemi F, Bazvand F, Makateb A. Lesions simulating
retinoblastoma at a tertiary care center. J Ophthalmic Vis Res
2015;10(3):316–19.
5. de Jong MC, de Graaf P, Noij DP, et al. Diagnostic performance of
magnetic resonance imaging and computed tomography for
advanced retinoblastoma: A systematic review and meta-analysis.
Ophthalmology 2014;121(5):1109–18.
6. Chawla B, Singh R. Recent advances and challenges in the
management of retinoblastoma. Indian J Ophthalmol 2017;65:133–9.
7. Broaddus E, Topham A, Singh AD. Incidence of retinoblastoma in
the USA: 1975‑2004. Br J Ophthalmol 2009;93:21–23.
8. MacCarthy A, Bayne AM, Draper GJ, et al. Non-ocular tumours
following retinoblastoma in Great Britain 1951 to 2004. Br J
Ophthalmol 2009;93:1159–62.
9. Austroads. Assessing fitness to drive. Sydney: Austroads, 2017.
Available at www.austroads.com.au/drivers-vehicles/assessing-
fitness-to-drive [Accessed 15 May 2018].
10. Fabian ID, Naeem Z, Stacey AW, et al. Long-term visual acuity,
strabismus, and nystagmus outcomes following multimodality
treatment in group D retinoblastoma eyes. Am J Ophthalmol
2017;179:137–44.
11. Batra A, Pushker N, Venkatesh P, Arora T, Tewari R, Bakhshi S.
Long-term visual outcomes in intraocular retinoblastoma with eye
preservation. Clin Transl Oncol 2016;18(10):1034–8.

9
Case 2 check Ophthalmology

CASE Question 3

2
What do the examination findings indicate?
Tom’s right eye is painful

Tom, a labourer aged 22 years, presents with a three-


day history of a non-specific pain in and around his right
eye. The eye has looked ‘angry’, with significant
hyperaemia. Initially the symptoms were mild, but they
have steadily worsened, and Tom feels his vision has
now become affected. Bright lights intensify the pain.
There has been no obvious discharge from the eye apart
from epiphora due to the light sensitivity. He cannot
recall any particular incident at the time the symptoms
began. He recalls having these symptoms a few years
ago, but the duration and intensity of the symptoms are
significantly greater now. The pupil also appears slightly
distorted in the right eye. Tom’s medical history is
unremarkable apart from a chronic sore lower back.
Question 4
What other information in Tom’s history is important to your
diagnosis?
Question 1
Describe the differential diagnoses for this presentation.

Question 5
Question 2
What is your diagnosis?
What signs or additional information or tests will help to
differentiate these conditions?

Further information Further information

On examination, Tom’s eye is severely injected around the You refer Tom for X-rays of his back and the results describe
limbal area. Anterior chamber cells are visualised on inflammation of the sacroiliac joints. Given his recurrent red
high magnification at the slit-lamp. eye, you also refer him to an ophthalmologist.

10
Ophthalmology check Case 2

Question 6 CASE 2 Answers


Are any further tests required?

Answer 1
The presentation of a unilateral, painful red eye may indicate
the presence of several conditions including:

• uveitis/iritis
• acute angle closure glaucoma
• corneal abrasion/ocular foreign body
• trauma
• conjunctivitis.
Although commonly bilateral in presentation, bacterial
conjunctivitis may present in one eye. The patient will
generally complain of a red, irritable eye. Purulent discharge is
Further information
the hallmark of a bacterial infection. The patient may
You request blood tests for HLA-B27. Tom’s results suggest complain of photophobia if there is associated punctate
that he is HLA-B27-positive. epitheliopathy reflecting a disrupted corneal (epithelial)
surface. Viral conjunctivitis can often present initially in one
eye. In some cases there can be photophobia and a drop in
vision due to inflammatory corneal infiltrates. The hallmark of
Question 7
viral conjunctivitis, however, is grittiness and watering.
How does this affect the outlook for Tom both in ocular and
Keratoconjunctivitis sicca (dry eyes) is a relatively common
general health terms?
condition. On presentation the patient will complain of
grittiness, epiphoria, photophobia and redness; symptoms
worsen throughout the day, and air conditioning and/or
heating will often exacerbate symptoms. The condition is
almost always bilateral on presentation and only in significant
cases would vision become affected.

An ocular foreign body will cause localised irritation and


inflammation. Patients commonly produce excessive tearing
as a result of the irritation. Similarly, the patient generally
complains of light sensitivity. Visual acuity may or may not be
affected, depending on the location of the foreign body and
the penetration within the eye. Often the patient can describe
an injury consistent with the onset of symptoms. It is
important to elicit on history the nature of the injury – if a
penetrating injury is suspected, immediate specialist
consultation is required.
Question 8
Acute angle-closure (AAC) is characterised by sudden and
What is the treatment for Tom’s ocular condition and general severe unilateral eye pain, red eye, poor vision and increase in
condition? intraocular pressure, and represents an ophthalmic
emergency. On presentation, the patient will complain of pain,
extreme photophobia and sudden deterioration of vision,
which may be significant. AAC occurs more commonly in
patients over the age of 55 years and in some ethnicities (eg
Asian).1 Females are also twice as likely to have an acute
attack.1 On examination, the pupil is semi-dilated and remains
unreactive to light; the cornea often looks hazy or cloudy.
Vision is affected by the presence of severe corneal oedema.

Uveitis is an intraocular inflammation that involves the uveal


tract (iris, ciliary body and choroid) and is often, but not
always, unilateral in presentation. Uveitis is classified
depending on the structures that are affected. Anterior uveitis

11
Case 2 check Ophthalmology

(or iritis), involving the iris and the anterior portion of the irregular-shaped and poorly reactive pupil) due to inflammation
ciliary body, develops over several days, beginning with is one of the most common complications of anterior uveitis.
general sensitivity around the eye and progressing to Patients with AAC often present with mid‑dilated pupils that
photophobia, redness and visual loss. Keratic precipitates, are unresponsive. However, pupil size or reactions may be
small cellular deposits resulting from anterior chamber difficult to assess in some cases because of the corneal haze. In
inflammation, may be seen on the posterior cornea (Figure 1). uveitis, keratic precipitates may be seen in the mid or inferior
Inflammation of the trigeminal nerve is common and leads to zone of the cornea. Small nodules (Koeppe nodules) may also
referred pain radiating around the eye area. Posterior uveitis is appear at the border of the pupil as a result of granulomatous
associated with inflammation of the posterior chamber. The uveitis. These are signs of chronicity.2
presenting symptoms of posterior uveitis are usually floaters
Fluorescein staining (under a blue or violet light) should indicate
and blurred vision. Floaters indicate vitreous inflammation,
corneal changes and is of use in cases of suspected foreign
whereas blurred vision may be due to associated macular
bodies. The examiner may notice disrupted or inadequate tear
oedema or retinal inflammation.
film and small erosions on the epithelial surface in cases of mild
dry eye. In severe cases, scarring may occur. Observation may
Answer 2
be difficult in patients with photophobia.
Patient history provides essential information for further
testing. Tom has no history of injury or purulent discharge to Answer 3
suggest either a foreign body or bacterial infection. The
description of pain is unlikely to reflect an acute rise in While classical teaching suggests that the location and
intraocular pressure, given that it did not have a sudden onset. intensity of hyperaemia may, in some patients, be helpful in the
It is also a more non-specific, vague sensation with differential diagnosis, clinically it can be very difficult to
photophobia and discomfort rather than the typical headache differentiate, especially without a slit lamp. Often, many layers
or brow ache associated with acute pressure rise. There is can be red and injected. Conjunctivitis often results in diffuse,
also no nausea or vomiting, which can accompany AAC. superficial redness with conjunctival oedema (chemosis). In
anterior uveitis, the injection may not be as diffuse and there
Visual acuity assessment can further help differentiate may be injection surrounding the limbus (ciliary injection). In
between conditions, although results may be less specific. cases of episcleritis, the injection is again more superficial, and
Mild dry eye and conjunctivitis are unlikely to affect visual instilling 10% phenylephrine may produce marked blanching
acuity significantly, although some variability may result from and almost complete disappearance of the injection. In scleritis,
excess tearing or discharge. An AAC attack will lead to however, even though the eye does blanch, deeper scleral
corneal oedema that will cause an abrupt decrease in vision. injection can remain after phenylephrine instillation.
Patients with significant corneal oedema may also describe
‘haloes’ around objects, which are due to the light scatter. Ciliary injection and the presence of keratic precipitates,
combined with the progression of symptoms including pain or
Observation is useful. In the general practice setting, this may globe tenderness, photophobia and blurred vision, suggest
be facilitated by using an ophthalmoscope with a ‘+’ setting to Tom has anterior uveitis.
act as a magnifying lens. Posterior synechiae (adhesion
between the anterior lens surface and iris resulting in an Uveitis accounts for approximately 10% of cases of total
blindness and affects approximately 1800 Australians each
year.3,4 Episodes of anterior uveitis account for 60–92% of all
uveitis diagnoses.3,4

Answer 4
In an active, young adult, back pain may incorrectly be
attributed to vigorous exercise or labour. Although acute
anterior uveitis may be idiopathic in origin, it may be
associated with an underlying autoimmune or systemic
condition.5 A thorough review of systems is important.
Systemic associations may include seronegative arthropathies
(such as ankylosing sponditis, psoriatic arthritis), inflammatory
bowel disease, sarcoidosis, syphilis and ocular tuberculosis.
The presence of anterior uveitis may serve to complete the
clinical picture for a collection of previously non-specific or
mild symptoms. In Tom’s case, further investigation of the
back pain is warranted.

Answer 5
Figure 1. Limbal injection due to anterior uveitis Inflammation of the spine and sacroiliac joints is a typical
symptom of ankylosing spondylitis. Inflammation causes

12
Ophthalmology check Case 2

stiffness and pain in the lower spine; these symptoms are Patients will need ongoing follow-up until resolution, which
worse on waking and relieved with activity. Other joints may may take weeks to months, depending on the severity of the
be affected by inflammation, as can other organs, such as the initial attack. Side effects of topical steroids include an
eyes, heart and lungs; 25–35% of patients with ankylosing increase in intraocular pressure, secondary glaucoma and
spondylitis will also have an attack of acute anterior uveitis at cataract formation, although the latter is unlikely to occur with
some stage.5 a short duration of treatment.

Coexisting signs can differentiate other systemic conditions In severe cases, systemic or periocular corticosteroids may be
that lead to inflammation of the spine. Reactive arthritis, which indicated, but should only be administered under the guidance
may follow an infection such as chlamydia, commonly displays of an ophthalmologist. Given the risk of side effects following
a triad of arthritis, urethritis and conjunctivitis. Skin lesions and steroid use, it is important to carefully screen and counsel
nail changes are associated with psoriatic arthritis. patients before treatment, as well as monitor them during
treatment. Generally, treatment is tapered over six weeks,
Answer 6 despite symptomatic and clinical quiescence, to avoid
bounce-back inflammation.
Spondyloarthropathies are commonly associated with the
presence of HLA-B27 antigen, with some variation in More recently, the use of a sustained-release dexamethasone
prevalence. Blood tests for the presence of the antigens are intravitreal implant has been used for posterior uveitis
not necessary to confirm diagnosis; however, the presence of episodes where corticosteroids are less effective through lack
antigens may provide information about the probability of of ocular penetration. The sustained-release mechanism has
future uveitis attacks. In recurrent episodes, particularly in been shown to be effective in treating residual inflammation
HLA-B27-negative patients, it is important to screen for and avoiding acute relapses.
underlying autoimmune disease, for example, by checking
Possible short-term complications of treatment include acute
antinuclear antibodies and extractable nuclear antibodies. An
intraocular pressure rise, which can lead to glaucoma and
infective origin (ie syphilis) may be considered in patients with
visual field loss over time. Posterior synechiae is common;
recurrent iritis and those considered to be high-risk patients.
therefore, long-acting cycloplegics such as homatropine and
atropine may be used to relieve pain and reduce the possibility
Answer 7
of adhesions. Patients with chronic disease are at greater risk
Over 90% of patients with ankylosing spondylitis are HLA‑B27- of complications such as cataract, ocular hypertension,
positive; the antigen represents the most significant association secondary glaucoma and cystoid macular edema.3
between uveitis and inflammatory systemic conditions.
Treatment of the systemic condition is dependent on
Conversely, a case of uveitis with a negative HLA‑B27 finding is
symptoms. Non-steroidal anti-inflammatory drugs may be
unlikely to be associated with systemic disease.5
used to treat sore or painful joints. If inflammation persists,
Specifically for ankylosing spondylitis, approximately 30% of oral or injectable corticosteroids or immunosuppressants and
patients will subsequently have an acute attack of uveitis. The biologics may be necessary. Care should be taken, given the
first episode of HLA-B27-positive uveitis often occurs at the possible side effects of these medications, and consultation
age of 20–40 years. Patients who are HLA-B27-negative are with a rheumatologist is essential. Physical therapy to improve
unlikely to have an attack until later. HLA-B27-positive uveitis flexibility and maintain correct posture is considered a useful
episodes tend to be unilateral and acute in the majority of adjunct to medical treatment in cases of ankylosing
patients and recur frequently when compared with HLA-B27- spondylitis.
negative patients, who are more likely to have bilateral,
chronic episodes (more than three months’ duration) with Resources for patients
minimal recurrence.5
• Royal Australian and New Zealand College of
HLA-B27-positive uveitis suggests a genetic predisposition, Ophthalmologists patient information sheet, https://
prompting the possibility of further familial investigation. ranzco.edu/ArticleDocuments/233/opa_ranzco_uveitis_
Patients who test positive for HLA-B27 and have symptoms and_iritis.pdf.aspx?Embed=Y
suggestive of ankylosing spondylitis are generally referred to a
rheumatologist for further assessment and investigation. Resources for doctors
• American Academy of Ophthalmology, http://eyewiki.org/
Answer 8
Acute_Anterior_Uveitis
The primary aim of ocular therapy remains the resolution of
• Rothova A, Buitenhuis HJ, Meenken, et al. Uveitis and
inflammation and prevention of vision loss. Referral to an
systemic disease. Br J Ophthalmol 1992;76(3):137–41.
optometrist or ophthalmologist within 48 hours is essential to
minimise short-term and long-term sequalae. • Wakefield D, Chang JH, Amjadi S, Maconochie Z, El-Asrar
A, McCluskey P. What is new HLA-B27 acute anterior
Corticosteroid eye drops, such as prednisolone, in addition to
uveitis? Ocul Immunol 2011;19(2):139–42.
a cycloplegic (eg atropine 1% or homatropine 2%) remain the
standard initial therapy for patients with anterior uveitis. This • Jabs DA, Nussenblatt RB, Rosenbaum JT. Standardization
should be done under specialist ophthalmic supervision. of uveitis nomenclature for reporting clinical data. Results

13
Case 2 check Ophthalmology

of the first international workshop. Am J Ophthalmol


2005;140(3):509–16. Available at www.ajo.com/article/
S0002-9394(05)00407-1/fulltext

References
1. Zhang X, Liu Y, Wang W, et al. Why does acute primary angle
closure happen? Potential risk factors for acute primary angle
closure. Surv Ophthalmol 2017;62(5):635–47. doi: 10.1016/j.
survophthal.2017.04.002.
2. Durrani OM, Tehrani NN, Marr JE, Moradi P, Stavrou P, Murray PI.
Degree, duration, and causes of visual loss in uveitis. Br J
Ophthalmol 2004;88(9):1159–62.
3. Zagora SL, Symes R, Yeung A, Yates W, Wakefield D, McCluskey
PJ. Etiology and clinical features of ocular inflammatory diseases
in a tertiary referral centre in Sydney, Australia. Ocul Immunol
Inflamm 2017;25(sup1):S107-S114.
4. Karaconji T, Maconochie Z, McCluskey P. Acute anterior uveitis in
Sydney. Ocul Immunol Inflamm 2013;21(2):108–114
5. Loh AR, Acharya NR. Incidence rates and risk factors for ocular
complications and vision loss in HLA-B27-associated uveitis. Am J
Ophthalmol 2010;150(4):534-542.e2.
6. Chang JH, McCluskey PJ, Wakefield D. Acute anterior uveitis and
HLA-B27. Surv Ophthalmol 2005;50(4):364–88.

14
Ophthalmology check Case 3

CASE Further information

3
Jack is just able to open his left eye to conduct a quick visual
Jack presents with sudden, assessment. He manages to see most letters on the 6/36 line
sharp eye pain on your chart.

Jack, aged 28 years, was working on a lathe when he felt


a sudden, sharp pain in his right eye. Immediately after
Question 4
the incident, he was unable to open the eye. His work
colleagues advised him to go the local general practice Are there tests or procedures that you should avoid during an
or hospital emergency department. He arrives at your examination?
practice with a makeshift patch across his eye and remains
in considerable pain.

Question 1
What signs may you expect to see on examination?

Question 5
What advice may be important for Jack prior to specialist
referral?

Question 2
What population groups typically present with penetrating eye
injuries? Where may these injuries occur?

Question 6
Prior to leaving your clinic, what can you do as a general
Question 3 practitioner (GP) to help Jack’s scenario?

What information may be helpful to your diagnosis?

15
Case 3 check Ophthalmology

Question 7 CASE 3 Answers


What are the basic aims of treatment for penetrating eye injuries?

Answer 1
Given that Jack had been working on a lathe and that he felt a
sudden, sharp pain, the most important aetiology to consider
first is the possibility of a penetrating eye injury. Although
relatively rare, penetrating eye injuries are often dramatic in
presentation. The extent of the injury will depend on the size,
location and level of penetration of the foreign object within
the eye.

Examination may be difficult if there is localised lid swelling. If


the eye can be observed directly, a number of signs may be
present. If the cornea has been lacerated, the front chamber of
Further information
the eye may appear flat because the aqueous fluid is lost
Jack was diagnosed with a full thickness corneal laceration due through the open wound. In severe cases, the iris may prolapse
to a metallic foreign object. He proceeded to surgery to repair through the wound itself. In the case of iris prolapse, the pupil
the open wound and re-establish the integrity of the eye. He will also appear distorted (teardrop appearance; Figure 1).
was told that follow-up surgery was likely, as the object had
penetrated both the iris and lens. Two weeks following surgical
intervention, visual acuity in the left eye is approximately 6/24.

Question 8
What is Jack’s prognosis for final visual rehabilitation?

Figure 1. Iris prolapse and hyphaemia following trauma

Further information
The shearing forces from the insult can rupture the blood
Eighteen months after the injury, Jack is still having some vessels within the anterior chamber of the eye, leading to
issues with glare and poor vision, despite being prescribed accumulation of blood in the front segment (hyphaema). There
corrective glasses. is often associated subconjunctival haemorrhage and/or
inflammation in the form of conjunctival chemosis. The
presence of any of these signs or a clinical suspicion of a
Question 9 penetrating eye injury warrants immediate referral to an eye
hospital emergency department. In general, ask the patient to
What issues may warrant additional surgery for Jack? Are
fast, in case surgery is required, and protect the eye with a
there other options to help Jack?
clear shield. Do not attempt to examine the eye further.
Enquire about the patient’s tetanus vaccination history and
inform the hospital that the patient is en route.

Answer 2
The incidence and cause of open globe injuries varies
considerably with age and location. Typically, penetrating
injuries occur within the workplace and affect young and
middle-aged men (21–40 years). Males account for two to
three times more injuries than women.1 Most foreign bodies
are metallic in nature. In older patients, falls account for the

16
Ophthalmology check Case 3

majority of open globe injuries.2 Previous intraocular surgery Answer 5


may serve to exacerbate the impact of these injuries.2 Ocular
It remains imperative not to apply pressure to the eye prior to
trauma in children often involves a sharp object and is more
specialist intervention. As surgery is considered likely,
likely to occur in the home environment. Consistent with all
particularly in severe cases, the patient should be advised not
ages, young boys have been reported to be twice as likely to
to drink or eat.
be affected as girls.3

Blunt ocular trauma injuries in sport are relatively common, but Answer 6
the penetrating injury is rare. In a recent review of ocular
A tetanus booster may also be appropriate in some
injuries, Australian researchers found that cycling and football
circumstances and can be given by the GP. Advise the
were the largest contributors to sports-related ocular hospital
patient to avoid unnecessary strain and movement as
admissions.4 Given the nature of cycling injuries, penetrating or
significant head movements can aggravate the condition;
projectile injuries were more likely to occur within this cohort.
this includes coughing or blowing their nose. An antiemetic
Approximately 20% of patients with severe facial trauma have may also be provided if the patient feels nauseous.
concurrent ophthalmic involvement, although these patients may
If safe to do so, prior to sending the patient to emergency, a
not commonly present to the standard general practice setting.5
clear eye shield may be placed over the eye to minimise direct
touch. If an eye shield is not available, a makeshift shield can
Answer 3
be fashioned from the end of a coffee cup (Figure 4), or the
The patient’s history remains essential, and this information patient could wear sunglasses.
may assist the operating surgeon if the foreign body is not
readily apparent. An attempt to assess visual acuity is
important; however, visual acuity is often non-specific,
although poor vision on presentation is often a factor in
determining the final visual outcome. Basic anterior
segment examination with a pen torch or ophthalmoscope
may help identify subtle injury (Figure 2). Fluorescein dye
may be used in conjunction with a cobalt blue light to
determine possible aqueous outflow through the cornea.
This would confirm the presence of a full-thickness
laceration (positive Seidel test; Figure 3).

Organising urgent specialist referral is paramount and should


be prioritised over vision and ocular assessment within the
general practice clinic.

Answer 4
If an object remains embedded within the eye, it should be left
in situ as extraction of the object may lead to herniation of the
ocular contents and exacerbation of the intraocular injury.
Similarly, intraocular pressure testing, either manually or Figure 3. Foreign body
digitally, should be avoided.

Figure 2. Positive Seidel test Figure 4. Makeshift eye shield

17
Case 3 check Ophthalmology

Answer 7 Corneal scarring may occur. Scarring located across the line
of sight will inhibit vision, and significant scarring may result
In cases of penetrating injury or full thickness laceration, the
in an irregular cornea, which is not fully correctable with
basic aim of surgery is to restore globe integrity. Removal of
glasses or soft contact lenses (Figure 6). Photophobia may
any intraocular foreign body is important as toxicity caused by
be a long-term consequence of an injury such as this and is
the object may occur. Failure to remove the foreign body may
best left without further intervention, because there is a risk
lead to further complications including, but not limited to,
of secondary infection. Corneal laser surgery
cataract formation, secondary glaucoma and retinal
(phototherapeutic keratectomy) to remove or reduce the scar
pigmentary degeneration.6 Immediate removal of the foreign
may be undertaken and has been successful in restoring
body may also reduce the possibility of proliferative retinal
vision.14 Corneal transplantation may be required in severe
complications.7 Post-traumatic endophthalmitis is a
cases of scarring in order to restore visual acuity. The
significant concern, and the early removal of the foreign body
possible long-term complications of surgery, including
(within 24 hours) in conjunction with systemic antibiotics will
rejection and corneal graft failure, must be considered prior
further reduce the risk of this complication.8,9
to undergoing this procedure.
If the patient is located in a rural area, they can be transferred
via road or air ambulance to the nearest specialist centre. At no Resources for doctors
time should an attempt to repair the ocular injury be attempted • American Academy of Ophthalmology, http://eyewiki.aao.org/
by anyone other than the ophthalmologist. Ocular_Trauma%3A_Acute_Evaluation,_Cataract,_Glaucoma

Answer 8
Given that Jack has had a serious injury and his current vision
remains relatively poor, the prognosis for visual recovery is
uncertain. Several prognostic factors have been identified as
being responsible for poor final visual acuity. These factors
include the size of the object and whether it was dirty or
soiled, the force of penetration, and delay in surgical
intervention. These increase the risk of retinal complications,
such as proliferative vitreoretinopathy and endophthalmitis.6,10
Reflecting the severity of the initial trauma, the presenting
visual acuity has been considered a more significant predictor
with respect to the final postoperative endpoint.2,6 Patients
seeing worse than 6/60 at presentation remain less likely to
achieve vision greater than 6/15.11 This level of vision will
affect standard daily tasks such as driving.

Answer 9
Given the damage to the lens at the incident, a traumatic
Figure 5. Post-corneal injury
cataract may develop in the eye. This will require removal to
provide visual rehabilitation. Traumatic cataracts develop in
50–74% of patients with intraocular foreign bodies.6,10 If
posterior segment surgery was undertaken at the time of injury,
cataract formation is expected within 12–18 months of the
procedure, regardless of the initial impact on the natural lens.12,13
The surgeon should inform the patient that secondary surgery is
likely at some stage.

Patients remain at risk of retinal detachment following


surgery and should be advised of possible symptoms (eg
flashes, floaters, increasing cloud across vision). This
represents a further ophthalmic emergency, although retinal
detachment remains a relatively rare consequence if the initial
ocular repair is successful.

If the iris has been damaged, the patient may continue to have
issues with glare and light sensitivity. Iris reconstruction may
be recommended if the patient is significantly affected by
symptoms. This may be performed manually or with the use
of secondary intraocular implants if the iris tissue is largely Figure 6. Corneal scarring post penetrating injury
absent (Figure 5).

18
Ophthalmology check Case 3

References
1. Batur M, Seven E, Esmer O, Akaltun MN, Yasar T, Cinal A.
Epidemiology of adult open globe injury. J Craniofac Surg
2016;27(7):1636–41.
2. Beshay N, Keay L, Dunn H, Kamalden TA, Hoskin AK, Watson SL.
The epidemiology of Open Globe Injuries presenting to a tertiary
referral eye hospital in Australia. Injury 2017;48(7):1348–54.
3. Sii F, Barry RJ, Abbott J, Blanch RJ, MacEwen CJ, Shah P. The UK
Paediatric Ocular Trauma Study 2 (POTS2): demographics and
mechanisms of injuries. Clin Ophthalmol 2018;12:105–11.
4. Hoskin AK, Yardley AM, Hanman K, Lam G, Mackey DA. Sports-
related eye and adnexal injuries in the Western Australian
paediatric population. Acta Ophthalmol 2016;94(6):e407–10.
5. Welman T, Shanmugarajah K, Sabah S, et al. Assessment of
emergency department eye examinations in patients presenting
with mid-face injury. J Emerg Med 2016;50(3):422–26.
6. Loporchio D, Mukkamala L, Gorukanti K, Zarbin M, Langer P,
Bhagat N. Intraocular foreign bodies: A review. Surv Ophthalmol
2016;61(5):582–96.
7. Yeh S, Colyer MH, Weichel ED. Current trends in the management
of intraocular foreign bodies. Curr Opin Ophthalmol
2008;19(3):225–33.
8. Essex RW, Yi Q, Charles PGP, Allen PJ.  Post-traumatic
endophthalmitis. Ophthalmology 2004;111(11):2015–22.
9. Kong GY, Henderson RH, Sandhu SS, Essex RW, Allen PJ,
Campbell WG. Wound-related complications and clinical
outcomes following open globe injury repair. Clin Exp Ophthalmol
2015;43(6):508–13.
10. Szijártó Z, Gaál V, Kovács B, Kuhn F. Prognosis of penetrating eye
injuries with posterior segment intraocular foreign body. Graefes
Arch Clin Exp Ophthalmol 2008;246(1):161–65.
11. Ehlers JP, Kunimoto DY, Ittoop S, Maguire JI, Ho AC, Regillo CD.
Metallic intraocular foreign bodies: characteristics, interventions,
and prognostic factors for visual outcome and globe survival. Am J
Ophthalmol 2008;146(3):427–33.
12. B
ai HQ, Yao L, Meng XX, Wang YX, Wang DB. Visual outcome
following intraocular foreign bodies: A retrospective review of
5-year clinical experience. Eur J Ophthalmol 2011;21(1):98–103.
13. Staffieri SE, Ruddle JB, Mackey DA. Rock, paper and scissors?
Traumatic paediatric cataract in Victoria 1992–2006. Clin Exp
Ophthalmol 2010;38(3):237–41.
14. Arfaj KA, Jain V, Hantera M, et al. Phototherapeutic keratectomy
outcomes in superficial corneal opacities. Ophthalmol Eye Dis
2011;16(3):1–6.

19
Case 4 check Ophthalmology

CASE Question 3

4 Meryl has unilateral


blurred vision
What is the significance of these findings?

Meryl, 67 years of age, presents to your practice


reporting a 10-day history of blurred vision in her left
eye. On further questioning, Meryl says she feels that
the visual field on her lower left side is depressed, and
she has also noticed increasing floaters in this eye.
She does not wear glasses for normal distance
activities; however, prior to cataract surgery three
years ago, she was significantly short-sighted in both
eyes. There is no history of recent trauma.

Question 1
What are differential diagnoses?

Question 4
Describe the possible mechanisms of retinal detachment.

Question 2
What tests can you perform in the clinic to help determine the
cause of Meryl’s symptoms?

Question 5
What are the possible risk factors for retinal detachment?

Further information

Meryl’s vision is 6/12 in the right eye and count fingers only in
the left. Visualisation through an ophthalmoscope is limited;
however, the red reflex appears diminished in the left eye,
compared with the other eye.

20
Ophthalmology check Case 4

Question 6 Question 9
When is referral necessary? What is the final visual prognosis?

Question 7
CASE 4 Answers
What are the treatment options?

Answer 1
Differential diagnoses can include:

• retinal tear or detachment


• vitreous haemorrhage
• posterior vitreous detachment
• diabetic vitreous haemorrhage
• migraine.
Vitreous gel fills the central cavity of the eye, providing
structural support. With age, the vitreous undergoes
Further information liquefaction, eventually leading to separation from the retina.
This is a relatively normal event and occurs in approximately
You refer Meryl for urgent assessment by an ophthalmologist.
60% of people aged 40–70 years. This process is called
Meryl undergoes surgery to repair her retinal detachment.
posterior vitreous detachment (PVD).2 As the vitreous pulls
Four days after surgery she is feeling generally unwell and has
away from the retinal layer, neurons are excited, leading to
been coughing. She attends your practice for assessment of
‘flashes’ seen by the patient. Symptoms are usually
these symptoms and asks your advice about her visual
intermittent and can occur over a period of time prior to full
recovery. Although her vision appears to be improving, she is
detachment from the inner layer. As the vitreous pulls away, it
worried that the coughing may cause damage to her eye.
is common for vitreous floaters to appear across the line of
sight. ‘Floaters’ represent collagen fibres within the gel or
previous attachment of the gel to the optic disc. With the
Question 8 increased mobility of the vitreous gel, the fibres may cross the
visual axis and lead to symptoms. The vast majority of PVDs
Is it likely that coughing will place Meryl at risk of further
occur spontaneously and are not related to stress, heavy
ocular damage?
exercise or trauma. Not all patients with a PVD will have
symptoms, and only a minority of cases proceed to a more
significant ocular condition.

If the vitreous is more firmly attached to the retina than the


retina is to the underlying pigment epithelial layer (retinal
pigment epithelium [RPE]), the PVD may lead to a tear or
detachment of the retina from the RPE layer. Until otherwise
proven, retinal detachment should be suspected in all cases of
flashes and floaters and treated as an ocular emergency.

Abnormal retinal blood vessels may be formed as a result of


ischaemia in diseases such as diabetic retinopathy. These

21
Case 4 check Ophthalmology

newly formed vessels lack endothelial tight junctions and, The distance at which the test is performed should be noted.
therefore, remain predisposed to spontaneous bleeding.
Confrontation field assessment can help identify gross field
Normal vitreous traction with eye movement is a further cause
loss. The patient is asked to cover one eye. Sitting opposite,
of rupture of these vessels. Symptoms of vitreous haemorrhage
the examiner closes their corresponding eye, providing their
are variable and may include painless unilateral floaters or
own visual field as a control. Keeping an equivalent distance
visual loss. Significant haemorrhage will affect acuity and visual
between the subject and examiner, one hand is slowly brought
fields. Patients often note that vision is worse in the morning as
in from the periphery until the patient can confidently identify
blood has settled to the back of the eye, covering the macula.
the location of the moving hand. Both arms should be
Migraine may affect the visual system in several distinct ways. extended to reduce any obvious bias (Figure 2).
Prior to the onset of the migraine, visual aura is common and
Ophthalmoscopy will often lead to diagnosis, although
approximately 40% of patients describe symptoms such as
distinguishing between differential diagnoses may be difficult
‘visual snow’ or ‘lightning flashes’ (Figure 1).1 Retinal migraines,
without training. Observing the retinal reflection (‘red reflex’)
although rarer, may lead to unilateral decreased or complete
is a simple way of confirming a gross abnormality in
vision loss. Ophthalmologic migraine may lead to temporary
ophthalmic emergencies. This can be performed at 50 cm
palsy of one or more cranial nerves involved in eye movement.
through an ophthalmoscope, with the patient directed toward
Symptoms generally resolve after the migraine episode.

Answer 2
The focus of examination by the general practitioner (GP)
should be to understand the visual impact and determine the
need for specialist referral.

Visual acuity with or without glasses is easily assessed and is


important for baseline and medico-legal reasons. If the patient
cannot see the chart, a subjective recording of counting
fingers, hand movements or even light perception is helpful.

Figure 2. Confrontation field

Figure 1. Aura associated with visual migraine Figure 3. Red reflex

22
Ophthalmology check Case 4

the light. The examiner should note the colour and any penetrating ocular trauma. Fibrous retinal tissue can attach
asymmetry between the reflexes of both eyes (Figure 3). itself to the vitreous gel, contracting the retinal layers and
leading to detachment. Pre-existing retinal degenerative
Answer 3 conditions can lead to the formation of potentially harmful
Vision remains a non-specific feature of general posterior eye fibrous areas of retina. Localised tumour growth can lead to a
disease; however, poor vision in retinal detachment suggests similar effect and is called exudative detachment.
central or macula involvement, which presents a reduced
prognosis for visual recovery. Poor vision demands immediate Answer 5
specialist referral.
Several factors can predispose to the onset of retinal
There is no way to distinguish a PVD from a retinal tear without a detachment. These include:
dilated retinal examination. The diminished left red reflex suggests
• Myopia – more than 40% of all patients with a retinal
an abnormal posterior segment or obstruction. A cataract may
detachment are myopic.2 Treating the myopia through
reduce the reflex; however, Meryl has had cataract surgery,
cataract or laser refractive surgery does not remove the risk
indicating the location of the condition at the posterior eye.
of retinal detachment. Patients most at risk are those with
longer axial lengths who have a spectacle correction of
Answer 4
–6.00 DS or greater.
Retinal detachment may result from several different
• Age – the incidence of retinal detachment increases with
processes including:
age because of the increased incidence of PVD.
• a break or tear in the retina (rhegmatogenous detachment)
• Surgery – can lead to increased vitreoretinal traction either
• uncontrolled diabetes causing a tractional retinal detachment. during or after the operation, particularly in the elderly.
Post-surgery rates of retinal detachment of up to 3% have
Rhegmatogenous is the most common type of detachment,
been described, albeit in complicated surgical cases.
with an incidence of between 6.3 and 17.9/100,000 and
significant variation in different ethnicities.2 Common causes • Trauma – both direct and indirect trauma to the eye may
include PVD, blunt trauma, intraocular surgery and myopia. lead to detachment. Contact sport is a risk factor, although
Rhegmatogenous tears occur most commonly in the superior falls in elderly patients remain responsible for a significant
temporal retinal periphery. Once the retina is torn, fluid from proportion of retinal detachment cases.3 Penetrating eye
the vitreous can enter the subretinal space and peel the retina injuries may lead to scarring and tractional complications.3
off. Initially, the detachment presents as a ‘curtain’ or shadow
• Systemic disease – several conditions, including Marfan’s
across the periphery. If left untreated, it will rapidly progress to
syndrome, Stickler’s syndrome, connective tissue disease
involve the macula and lead to severe loss of vision (Figure 4).
and uncontrolled diabetes mellitus, confer a higher risk of
retinal detachment. Patients with these conditions should
undergo regular ocular examinations.

Answer 6
Retinal detachment represents a serious ocular emergency, and
patients should be referred immediately to an ophthalmologist
for further assessment, especially if visual acuity is reduced.

Answer 7
In the majority of cases, surgery is indicated to restore retinal
anatomy with the goal of restoring vision. If the macula is
involved in the retinal detachment, then the visual outcome
is most successful if done within a 24-hour period. If the
macula has been detached for a significant duration, then
surgery is unlikely to result in a positive outcome. The
surgical approach will depend on the type, underlying cause,
location and size of the detachment. Inflammatory
detachments may only require drainage of the subretinal
Figure 4. Severe retinal detachment (note: the green colour fluid. In mild tears without subretinal fluid, a laser may be
represents an image artefact) used to produce a series of small burns to surround the tear.
The process leads to formation of scar tissue, which
effectively seals the retina to the underlying tissue. In this
Tractional detachments occur as a result of adhesions between case, treatment is painless and can be done in the specialist
the vitreous and the retina. The most common causes of rooms under topical anaesthesia.
tractional detachment are proliferative diabetic retinopathy and
If laser is not easily performed because of concurrent ocular

23
Case 4 check Ophthalmology

conditions (eg small, non-dilating pupil or very peripherally


Resources for doctors
located tear), cryotherapy may be used in a similar way. If the
tear has progressed to a full detachment, laser will have no • American Academy of Ophthalmology, http://eyewiki.org/
effect and surgical repair is required. This may include a Retinal_Detachment
vitrectomy and the use of intraocular gas to massage the
detached layer back to its original position (pneumatic References
retinopexy) or externally suturing a small silicon band or 1. Marzoli SB, Criscuoli A. The role of visual system in migraine.
Neurol Sci 2017;38(Suppl 1):99–102.
buckle onto the sclera (scleral buckle), pushing the detached
area back onto the retina. Laser or cryotherapy may be used to 2. Mitry D, Charteris DG, Fleck BW, Campbell H, Singh J. The
epidemiology of rhegmatogenous retinal detachment:
support these methods and is applied around retinal tears and geographical variation and clinical associations. Br J Ophthalmol
breaks. Gas inserted into the posterior segment to aid the 2010;94(6):678–84. doi: 10.1136/bjo.2009.157727.
re-attachment absorbs slowly and delays visual recovery. 3. Beshay N, Keay L, Dunn H, Kamalden TA, Hoskin AK, Watson SL.
The epidemiology of Open Globe Injuries presenting to a tertiary
Answer 8 referral eye hospital in Australia. Injury 2017;48(7):1348-1354. doi:
10.1016/j.injury.2017.04.035.
In most cases the patient may quickly resume mild activities 4. Ross WH. Visual recovery after macula-off retinal detachment. Eye
following surgery. Bending over or lifting heavy objects (Lond) 2002;16(4):440–46.
(0.5−10 kg), however, should be avoided. Coughing or 5. Ugarte M, Williamson TH. Horizontal and vertical micropsia
straining following surgery rarely leads to further following macula-off rhegmatogenous retinal-detachment surgical
complications; however, the patient should be advised about repair. Graefes Arch Clin Exp Ophthalmol 2006;244(11):1545–48.
the recurrence of ocular symptoms and to seek specialist
attention if this occurs. If gas is used, the surgeon may
require that the patient maintain a specified head position in
the immediate postoperative period. If an expansile gas is
used, air travel and even driving to areas of high altitude
need to be avoided until the gas has dissipated from the eye,
as the gas can expand and cause a significant rise in the
intraocular pressure. Patients should consult their surgical
team for specific instructions in this regard.

Answer 9
Visual prognosis is dependent on the duration, underlying
cause and location of the detachment. Although the majority
of surgical procedures successfully re-attach the retinal layer,
if the macula itself has been detached, particularly for an
extended period (>7–10 days), then vision is unlikely to return
to pre-detachment level.

Approximately 75% of patients will achieve best corrected


vision of 6/12 or better, which represents the legal limit of
driving within Australia.4 Macular detachment may lead to
long-term central visual distortion, even following
successful reattachment.5

A peripheral tear or detachment does not affect the central


vision. This is usually amenable to laser treatment in the
specialist rooms and does not require surgery. However, there
is always a risk that a secondary detachment may occur, so
the patient must be warned that if their symptoms change
(floaters, photopsia or field defects), they should consult an
eye care professional immediately.

Early detection and referral are the most important factors in


achieving a good visual outcome.

Resources for patients


• Royal Australian and New Zealand College of
Ophthalmologists patient information sheet, https://
ranzco.edu/ArticleDocuments/233/opa_ranzco_floaters_
and_flashes.pdf.aspx?Embed=Y

24
Ophthalmology check Case 5

CASE Question 2

5
What do you need for examination of Danny’s eyes? What will
Danny has itchy eyes you look for?

Danny, eight years of age, is brought in by his


aunty Dora. He comes from a remote community
in northern Western Australia and is visiting
family for a few months. Dora is your regular
patient; she is an Aboriginal woman with a
young family of her own, Shane (6 years) and
Ayisha (8 months).

Danny is shy. Dora does the talking for him. He


has had sticky eyes in the morning for the past
few weeks since he has been staying with Dora.
Danny’s eyes are not sore and his eyesight
seems fine.
Further information

On examination, Danny’s eyes seem normal and there is no


discharge. His uncorrected visual acuity is 6/6 in both eyes.
Question 1 On eversion of the eyelids you see seven to eight faint white
swellings scattered over the tarsal plate, and the everted
How would you approach this consultation?
surface looks red (Figure 1).

Further information Figure 1. Everted surface

Danny tells you he is from a small community near


Fitzroy Crossing. His mum came down with him for a
funeral in your town and asked Dora if he could stay with Question 3
her. He misses playing footy with his brothers and
What is the most likely diagnosis? What causes this condition?
friends. He cannot think of anyone in the family at home
with sticky eyes.

His eyes have been sticky ‘for a long time’. They are a bit
itchy in the mornings, but after he has washed his face,
they are fine for the rest of the day. He is otherwise well.
He is not on any medications, does not use eyedrops and
has no allergies. Dora has started him at the same school
as her son and he says he likes school.

Dora reports that he is settling in well with the family and


loves playing with her children. Ayisha has also
developed sticky eyes over the past week. She is worried
that Ayisha has caught something from Danny.

25
Case 5 check Ophthalmology

Question 4 1.5 cm above the eyelash margin, so that it acts as a


fulcrum on the eyelid. Pull the eyelashes gently and
How would you manage Danny and his household?
firmly out and up to evert the lid (everting the eyelid is
shown at www.youtube.com/watch?v=1RBoDT1zCh4).

The normal conjunctiva is pink, smooth, thin and transparent, and


blood vessels should be seen running vertically over the area.1

Answer 3
Danny has trachoma. According to the WHO trachoma
grading card definitions, Danny meets the criteria for follicular
trachoma, with more than five follicles over the tarsal plate.1
Trachoma is a clinical diagnosis, so no swabs are necessary,
except to check for a bacterial infection if there is profuse
purulent discharge present. Other differential diagnoses
include viral conjunctivitis (usually less than one-week
duration) and allergic conjunctivitis.

Trachoma is caused by Chlamydia trachomatis serotypes


A–K.2 Repeated scarring and contraction of the eyelids from
this infection ultimately result in inturned eyelashes. The
CASE 5 Answers
misdirected eyelashes rub on the eyeball (trichiasis), causing
intense pain and resulting in blindness if untreated.
Answer 1
Unlike genital trachoma, which is usually sexually transmitted,
First, try to establish rapport with Danny. You could start by ocular trachoma is spread between close household contacts,
asking him some general questions to initiate conversation by contact with infected secretions, towels, washers and flies.
and encourage him to speak to you. For example: As a result of reductions in overcrowding, improved access to
clean water and good hygiene, trachoma has disappeared
• Where are you from?
from much of the developed world, including mainstream
• How many houses are there in your community? populations in Australia.3 However, while the rates of blinding
trachoma have fallen over the past five years,4 Australia is the
• Who else lives in your house?
only developed country in the world where endemic levels of
Then ask specific questions about his eyes: trachoma still exist, affecting two-thirds of remote Aboriginal
communities.5
• How much discharge is there?
It was estimated in 2002 that 3.6% of blindness worldwide
• Does the discharge glue his eyes together?
was due to trachoma.6 The Global Elimination of Trachoma by
• Have any drops been used to treat the discharge? 2020, or GET2020, is a campaign initiated by the WHO to
address trachoma. It uses the acronym SAFE (Surgery for
• Has anyone else who usually lives with Danny had sore or
trichiasis, Antibiotics, Face washing, Environmental
sticky eyes?
improvements). In remote Australia, where trachoma is
• Has anyone in Dora’s household had sore or sticky eyes? present, annual surveys are still performed to assess
prevalence in households. As well as treatment of individual
Answer 2 cases, depending on certain prevalence thresholds, whole
households or entire communities may be treated with oral
It is important that the patient is relaxed when conducting the
antibiotics to decrease overall infection rates.
examination. A light cotton bud and a magnifying glass are
also helpful. The World Health Organization (WHO) have
Answer 4
produced a grading card, available on their website, that
provides a step-by-step approach to the examination (refer to Danny’s ocular trachoma can be treated with oral azithromycin
Resources for doctors).1 20 mg/kg as a stat dose.7 This antibiotic is available as an oral
suspension in banana and cherry flavours. Given Ayisha now
Look for obvious discharge and whether the eyes are normal,
has symptoms, it would also be worth examining and treating
and assess visual acuity (with pinhole if required):1
her and examining Dora’s other son. You should ask Dora to
• examine the eyelids, looking for inturned eyelashes monitor the whole family for any further symptoms and to
encourage daily hand and face washing.
• examine the cornea and note if there is any opacity
It is likely that Danny’s family in his original community will
• evert the upper eyelid and examine the conjunctiva
be rapidly re-infected if they are treated outside of a
–– to evert the eyelid, ask the patient to raise their chin but community-based screening program. It is not helpful,
look down, then position the cotton bud horizontally, therefore, to initiate their contact tracing and treatment.8 A

26
Ophthalmology check Case 5

call to their nearest public health unit may be useful to


ascertain if there is regular screening scheduled. Danny
should be treated outside of this screening, as he is staying
for a period with Dora’s family, and this will stop the spread
of infection within his current household.

Resources for Doctors


• Trachoma Grading Card WHO, www.who.int/blindness/
publications/trachoma_english.jpg?ua=1

• Australian Guidelines for the Public Health Management of


trachoma, www.health.gov.au/internet/main/publishing.
nsf/Content/cda-cdna-pubs-trachoma.htm

References
1. World Health Organization/Department of control of neglected
tropical diseases. Trachoma Grading Card. Geneva: WHO, 1987.
Available at www.who.int/blindness/publications/trachoma_
english.jpg?ua=1 [Accessed 21 March 2018].
2. Bashour, M. Ophthalmic manifestations of chlamydia. Medscape,
2016. Available at https://reference.medscape.com/
article/1203385-overview [Accessed 21 March 2018].
3. The National Trachoma Surveillance and Reporting Unit.
Australian Trachoma Surveillance Report 2012: Results–national
results 2012. NSW: The Kirby Institute for infection and immunity
in society, 2013. Available at www.health.gov.au/internet/
publications/publishing.nsf/Content/2012-trachoma-
toc~results~national-results [Accessed 21 March 2018].
4. Melbourne School of Population and Global Health. The
Indigenous Eye Health Roadmap. Melbourne, Vic: The University
of Melbourne, 2012 [updated 2017]. Available at https://mspgh.
unimelb.edu.au/centres-institutes/centre-for-health-equity/
research-group/ieh#roadmap [Accessed 21 March 2018].
5. Vision 2020 Australia. Indigenous eye health. Melbourne, Vic:
Vision 2020 Australia, [date unknown]. Available at www.
vision2020australia.org.au/our-work/indigenous-eye-health
[Accessed 21 March 2018].
6. World Health Organization. Prevention of blindness and visual
impairment: Causes of blindness and visual impairment. Geneva:
WHO, [date unknown]. Available at www.who.int/blindness/
causes/en/ [Accessed 21 March 2018].
7. Rostami, S. Trachoma. Medscape, 2016. Available at https://
emedicine.medscape.com/article/1202088 [Accessed 21 March
2018].
8. Communicable Diseases Network Australia. Guidelines for the
public health management of trachoma in Australia. Canberra:
CDNA, 2014. Available at www.health.gov.au/internet/main/
publishing.nsf/Content/cda-cdna-pubs-trachoma.htm [Accessed
21 March 2018].

27
Case 6 check Ophthalmology

CASE noted, with bilateral macular lesions, dot haemorrhages,

6
exudate and blood vessel tortuosity. The optometrist refers
her to an ophthalmologist and sends you a copy of the
Cynthia has type 2 diabetes examination findings. The optometrist also advises Cynthia to
see you again for review of her diabetes management.
Cynthia, a librarian aged 50 years, is one of your
regular patients and was diagnosed with type 2
Question 2
diabetes mellitus (T2DM) two years ago. Initially,
Cynthia has been able to control her blood glucose What do the results of Cynthia’s eye examination indicate?
levels with lifestyle modification, including exercise and
dietary modification. She has never smoked and drinks
alcoholic beverages only occasionally. She has no other
medical conditions and is not on any medications.

She presents complaining of increased blurred vision


over the past six months, especially when reading or
watching television. She states her work duties are
becoming more difficult to perform.

Question 1
What would be your first course of action?

Question 3
What are the risk factors for diabetic retinopathy? How can
these be minimised?

Further information

Cynthia tells you that in the past year, her glycated


haemoglobin (HbA1c) levels have tended to be above target
levels, despite exercising regularly and being careful with her
diet. She is reluctant to take medications but is also Question 4
concerned about developing complications of diabetes. She
How is the severity of diabetic retinopathy assessed? What is
agrees to starting metformin therapy.
Cynthia’s current level of retinopathy?
She makes an appointment to see an optometrist for an eye
examination later that day. Fundus examination and retinal
photography show no abnormal changes, but visual
assessments shows age-related deterioration in near vision,
and Cynthia is prescribed reading glasses.

Over the next two years, Cynthia’s vision deteriorates


further, and she sees the optometrist for her next two-year
appointment. She admits that she has not been taking her
hypoglycaemic medication and has not kept up her
exercise program.

Visual acuity, with her glasses script (moderate level myopia


with low-level astigmatism in each eye), is currently R: 6/36
L: 6/18. On ocular fundus examination, a hazy fundus view is

28
Ophthalmology check Case 6

photography may also be performed, but this will not include


Question 5
a proper examination of the retinal periphery. An item on the
How is diabetic retinopathy managed? Medicare Benefits Schedule (MBS) for retinal photography
with a non-mydriatic retinal camera is now available for
general practice use.2

Answer 2
The results of Cynthia’s eye examination indicate that Cynthia has
diabetic retinopathy and macula oedema. In Australia, 25–35% of
people with diabetes have some degree of diabetic retinopathy
and about 65,000 develop sight-threatening eye disease.4 It is the
leading cause of preventable blindness in Australia.4 The
prevalence of diabetic retinopathy is twofold to fourfold higher in
Aboriginal and Torres Strait Islander peoples than in non-
Indigenous communities, and annual screening is recommended
for Aboriginal and Torres Strait Islander patients with diabetes.5
Further information
Diabetic retinopathy manifests initially as non-proliferative
Cynthia saw her ophthalmologist, who confirmed that she has (NPDR), but can develop into proliferative (PDR). NPDR results
Stage 3 NPDR and commenced treatment for her CSME with from diabetes-related changes in the retinal microvasculature
monthly anti-VEGF injections for six months. She is now and can include microaneurysms; dot, blot or flame
being monitored. Her ophthalmologist also advised that she haemorrhages; exudates; microvascular abnormalities and
consult with you, her general practitioner, with regards to venous beading.5 PDR occurs in response to retinal ischaemia
commencing fenofibrate therapy, which you have now and is characterised by the development of new retinal blood
instituted. She has also seen you for review of her diabetes. vessels and fibrous tissue.5 Diabetic macula oedema can occur
She says that she appreciates the advice and education you’ve at any stage. The oedema is due to capillary leak in the macula
provided about managing her blood glucose levels. She has or perimacular region and causes thickening of the retina. If
resumed her exercise program and is adhering to her present within or close to the central macula, it is termed
medication regimens. She sees the optometrist once a year clinically significant macula oedema (CSME) affect, which is
and ophthalmologist on referral by her optometrist. She is associated with decreased central vision.5 It is often seen
pleased that her retinopathy has not progressed. concurrently with NPDR and is the most debilitating cause of
vision loss. About one-third of patients with diabetic
neuropathy develop CSME and, of these, 40% may become
legally blind.
CASE 6 Answers
Answer 3
Answer 1 All people with diabetes mellitus are at risk of developing
Cynthia’s blood glucose and HbA1c levels should be checked to retinopathy. The duration of diabetes is the main factor
ensure they are being maintained within normal ranges. Tight determining the prevalence of retinopathy. The key factors
control of blood glucose is essential and will help to reduce or associated with increased risk are:
delay the onset of diabetes complications, such as diabetic eye
• hyperglycaemia
disease.1 You may need to consider commencing Cynthia on an
oral hypoglycaemic agent if her blood glucose and HbA1c levels • hypertension
are not being adequately managed with lifestyle modification.
• dyslipidaemia.
The recommended HbA1c target for people with T2DM is ≤53
mmol/mol (≤7%);2 however, given variation in laboratory Optimising glycaemic and blood pressure control reduces the
results, an HbA1c level in the range of 48–58 mmol/mol (6.5– risk of developing diabetic retinopathy and slows its
7.5%) is recommended.2 progression in those with signs of retinopathy.1,6,7

Patients with T2DM are at risk of developing microvascular


Answer 4
disease that can result in complications such as diabetic
retinopathy. Regular eye examinations are essential, as early A grading system has been devised to identify the main stages in
detection of abnormal changes can reduce the risk of serious, the progression of diabetic retinopathy. This system is the
sight-threatening consequences.2,3 An eye examination, International Clinical Disease Severity Scale for Diabetic
consisting of a dilated fundus examination and assessment of Retinopathy and classifies diabetic retinopathy into five stages:5,8
visual acuity, should be done at baseline at the time of
• Stage 1: no apparent retinopathy
diagnosing. 2 Following this initial assessment, the patient
should be referred to an optometrist or ophthalmologist for • Stage 2: mild NPDR, characterised by the presence of a few
screening and follow-up every two years.2 Retinal microaneurysms

29
Case 6 check Ophthalmology

• Stage 3: moderate NPDR, characterised by a greater treatment), as well as ongoing intravitreal anti-VEGF and/or
number of microaneurysms and intraregional corticosteroid treatment.4,10 Laser photocoagulation is used to
haemorrhages than seen at Stage 2 produce laser burns in areas of the retina away from the
macula, with the aims of causing shrinkage of abnormal new
• Stage 4: severe NPDR, characterised by haemorrhages
blood vessels, which are usually fragile and prone to leaking,
across the entire retina or venous beading or intraretinal
and preventing further deterioration of vision. Vitrectomy is
microvascular abnormalities
the surgical removal of vitreous gel in the centre of the eye
• Stage 5: PDR, characterised by neovascularisation of the and is used to treat severe bleeding into the vitreous.12
disc or elsewhere (eg retina, iris), and can be complicated by
Management of low vision
vitreous haemorrhage or tractional retinal detachment.
Low vision affects patients in different ways. In general, patients
At each of the five stages, the presence of macula oedema is
with low vision have difficulty with activities of daily living and
also assessed.5
may be unable to maintain paid employment. Depending on the
Cynthia’s results are consistent with Stage 3 retinopathy level of vision loss, they may not be eligible to drive.
(moderate NPDR) with CSME.
Low vision also increases the risk of falls – patients with low
Answer 5 vision are twice as likely to fall and four to eight times more
likely to have hip fractures. They are also three times more
Management of risk factors likely to have depression through loss of independence and
Tight glycaemic and blood pressure control, and regular decrease in socialisation activities.13
monitoring of patients with diabetes can prevent serious and Vision Australia provides comprehensive services for people with
sight-threatening eye disease. low vision or blindness (refer to ‘Resources for patients and
Monitoring doctors’). Initial assessments are provided at no cost to the
patient and can be performed in the patient’s home or workplace.
Patients diagnosed with diabetic retinopathy should be
monitored at least every 12 months or more frequently, Conclusion
depending on the disease level and at the discretion of their
treating ophthalmologist, who will often co-manage these Patients with diabetes are at risk of developing vision-
patients with optometrists.4 threatening complications such as CSME. Diabetic retinopathy
is the leading cause of preventable blindness in Australia.4
Treatment of early-stage disease However, tight glycaemic control and regular monitoring of
Treatment of CSME and NPDR may include a combination of patients, including annual eye examinations, can result in early
laser photocoagulation, use of antibodies against vascular detection of subtle retinopathy and commencement of
endothelial growth factor (VEGF) intravitreal injections and/or treatment, if necessary, with a view to preventing vision loss.
intravitreal corticosteroids.
Resources for patients and doctors
Anti-VEGF treatment is quite recent – in use only in the past
10 years – and has been used very successfully for the • Vision Australia, www.visionaustralia.org/
treatment of diabetic macula oedema by intravitreal injection.
These antibodies prevent neovascularisation and leakage References
from blood vessels, which is the underlying disease process. 1. Holman RR, Paul SK, Bethel A, Matthews DR, Neil AW. 10-year
Anti-VEGF drugs currently available are bevacizumab, follow-up of intensive glucose control in type 2 diabetes. N Eng J
Med 2008;359(15):1577–89. doi: 10.1056/NEJMoa0806470.
ranibizumab and aflibercept. The safety and efficacy of
2. The Royal Australian College of General Practitioners. General
ranibizumab and aflibercept are very similar in the treatment
practice management of type 2 diabetes. Melbourne: RACGP,
of this disease, and studies have shown that the rate of 2016. Available at https://www.racgp.org.au/your-practice/
systemic side effects is low. guidelines/diabetes [Accessed 22 May 2016].

With respect to systemic treatment, recent studies have 3. World Health Organization. Causes of blindness and visual
impairment. Geneva: WHO, 2018. Available at www.who.int/
shown that fenofibrate, a lipid-lowering drug, reduces the risk
blindness/causes/en [Accessed 22 May 2018].
of development of diabetic retinopathy and progression to
4. Macular Disease Foundation Australia. Diabetic eye disease.
proliferative diabetic retiopathy, and reduces the treatment Sydney: Macular Disease Foundation, 2018. Available at www.
burden of laser photocoagulation.9–11 The benefits of mdfoundation.com.au/sites/default/files/MDFA_
fenofibrate on retinopathy were seen in patients with and DiabeticBooklet%202017-03_Web_0.pdf [Accessed 22 May 2018].
without dyslipidaemia. In Australia, the Therapeutic Goods 5. National Health and Medical Research Council. Guidelines for
Administration has approved the use of fenofibrate in patients management of diabetic retinopathy. Canberra: Commonwealth of
Australia, 2008. Available at www.nhmrc.gov.au/_files_nhmrc/
with type 2 diabetes and existing diabetic retinopathy.
publications/attachments/di15.pdf [Accessed 22 May 2018].
Treatment of advanced disease 6. Diabetes Control and Complications Trial Research Group. The
effect of intensive therapy of diabetes on the development and
The ocular treatment options for PRD are more likely to progression of long-term complications in insulindependent
include surgical intervention (vitrectomy surgery and laser diabetes mellitus. N Engl J Med 1993;329:977–86.

30
Ophthalmology check Case 6

7. Ferris FL, Nathan DM. Preventing diabetic retinopathy progression.


Am Acad Ophthalmol 2016;123(9):1840–42.
8. Wu L, Fernandez-Loaiza P, Sauma J, Hernandez-Bogantes E,
Masis M. Classification of diabetic retinopathy and macula edema.
World J Diab 2013;4(6):290–94.
9. Chew EY, Davis MD, Danis RP, et al. The effects of medical
management on the progression of diabetic retinopathy in persons
with type 2 diabetes: the Action to Control Cardiovascular Risk in
Diabetes (ACCORD) eye study. Ophthalmology 2014;121:2443–51.
10. Keech A, Simes RJ, Barter P, et al. FIELD study investigators.
Effect of long–term fenofibrate therapy on cardiovascular events in
9795 people with type 2 diabetes mellitus (the FIELD study):
randomised controlled trial. Lancet 2005;366:1849–61.
11. Sharma N, Ooi, JL, Ong J, Newman D. The use of fenofibrate in the
management of patients with diabetic retinopathy: An evidence-
based review. Aust Fam Physician 2015;44(6):367–70. Available at
www.racgp.org.au/afp/2015/june/the-use-of-fenofibrate-in-the-
management-of-patients-with-diabetic-retinopathy-an-evidence-
based-review/#10 [Accessed 22 May 2018].
12. National Eye Institute. Facts about diabetic eye disease.
Bethesday, MD: National Institutes of Health, 2018. Available at
https://nei.nih.gov/health/diabetic/retinopathy [Accessed 22
May 2018].
13. C
entre for Eye Research Australia. Clear Insight: The economic
impact and cost of vision loss in Australia. Melbourne: Centre for
Eye Research Australia, 2004.

31
Multiple choice question check Ophthalmology

Further information
ACTIVITY ID 135931
You refer Sally-Anne to an ophthalmologist, who confirms the
Opthalmology diagnosis of early-stage retinoblastoma with no vitreous
involvement or penetration into the extraocular tissues.
This unit of check is approved for six Category 2
points in the RACGP QI&CPD program. The Question 2
expected time to complete this activity is three
In Sally-Anne’s case the treatment of choice would be:
hours and consists of:
A. selective intra-arterial chemotherapy (SIAC)
• reading and completing the questions for each
case study B. intravitreal chemotherapy
–– you can do this on hard copy or by logging on C. enucleation
to the gplearning website, http://gplearning.
D. radiotherapy.
racgp.org.au

• answering the following multiple choice questions Case 2 – Colleen


(MCQs) by logging on to the gplearning website,
http://gplearning.racgp.org.au Colleen, 58 years of age, presents with sudden severe pain
and decreased vision in her left eye. The eye is red and
–– you must score ≥80% before you can mark the sensitive to light. Colleen tells you that she also feels
activity as ‘Complete’ nauseous. On examination, the pupil is unreactive to light.
• completing the online evaluation form.
Question 3
You can only qualify for QI&CPD points by
completing the MCQs online; we cannot process The most likely diagnosis is:
hard copy answers. A. anterior uveitis
If you have any technical issues accessing this B. conjunctivitis
activity online, please contact the gplearning
helpdesk on 1800 284 789. C. keratoconjunctivitis sicca

If you are not an RACGP member and would like to D. acute angle closure.
access the check program, please contact the
gplearning helpdesk on 1800 284 789 to purchase Question 4
access to the program. How will you manage Colleen?

A. Check for the presence of an ocular foreign body.

Case 1 – Sally-Anne B. Check for signs of a bacterial infection.

Gillian brings her daughter Sally-Anne, three years of age, to C. Refer her immediately to an eye hospital emergency
see you. Gillian is worried because Sally-Anne’s right pupil department.
seems to have a white reflection. When asked about her D. Assess her visual acuity to determine the extent of vision loss.
daughter’s history, Gillian tells you that Sally-Anne was born
at term. She weighed about 3 kg at birth and there were no Case 3 – Dean
antenatal or perinatal concerns. Her growth and development
have been normal. Gillian also tells you that neither she nor Dean, 25 years of age, presents with an eye injury. He was
Sally-Anne’s father have any medical or family history of eye mowing the lawn when he felt a sudden, sharp pain in his left
disorders. On examination, Sally-Anne has a white reflex and eye. He is unable to open his left eye and is worried that
you notice some small cream-coloured nodular masses. There something sharp may have penetrated his eye while he was
is no opacification of the lens. mowing. You organise urgent specialist referral for Dean.

Question 1 Question 5
Given the history and examination findings, the most likely When managing a penetrating eye injury in the general
diagnosis to consider is: practice setting, it is important to:

A. persistent hyperplastic primary vitreous (PHPV) A. avoid applying pressure to the eye

B. congenital cataract B. extract any object that remains embedded in the eye

C. retinoblastoma C. assess intraocular pressure

D. retinal detachment. D. administer antibiotic drops to protect against infection.

32
Ophthalmology check Multiple choice question

Case 4 – Celine Question 9


Celine, 59 years of age, presents with blurred vision and Stage 2 retinopathy is:
complains of flashes and floaters in her right eye. The
A. mild non-proliferative diabetic retinopathy (NPDR),
symptoms started three days ago, after she tripped and fell on
characterised by the presence of a few microaneurysms
the footpath when walking home from the bus stop.
B. proliferative diabetic retinopathy (PDR), characterised by
Question 6 neovascularization of the disc (NVD)

What is the most important diagnosis to consider in C. characterised by the presence of macula oedema
Celine’s case?
D. characterised by the presence of haemorrhages across the
A. Posterior vitreous detachment entire retina.

B. Retinal detachment
Question 10
C. Migraine with aura
To prevent progression of his retinopathy, Colin should be
D. Postural hypotension advised to:

A. maintain tight glycaemic control with lifestyle measures


Case 5 – Ariel
and, if necessary, glucose-lowering medication
Ariel, eight years of age, comes to see you with her mother.
B. maintain blood pressure control
Ariel has been complaining of ‘itchy, sticky eyes’ for the past
two weeks. Ariel lives with her parents and two brothers in a C. have eye tests at least every 12 months
remote community in the Northern Territory, but they have
D. do all of the above.
recently moved closer to the city. You examine Ariel’s eyes and
diagnose follicular trachoma.

Question 7
The criteria for a diagnosis of follicular trachoma is the
presence of:

A. purulent discharge

B. five or more follicles over the tarsal plate

C. inturned eyelashes

D. all of the above.

Question 8
Management of this consultation should include:

A. treating the trachoma with oral azithromycin 20 mg/kg as


a stat dose

B. encouraging regular hand and face washing

C. advising Ariel’s mother to monitor the rest of the family


and seek treatment if symptoms develop.

D. all of the above.

Case 6 – Colin
Colin, 48 years of age, has type 2 diabetes mellitus (T2DM),
which was diagnosed five years ago. Colin’s last eye
examination, one week ago, showed signs of Stage 2
retinopathy.

33
How Vision Australia
can support your patients

Vision loss Choice of 28 150+ years of Specialists


assessment national service understanding in technology
& services locations and service to the training adapted
from 44+ allied or choose an blind & low vision for all ages
health professionals in-home visit community

Build freedom Achieve Emotional support Extensive


and safety aspirations groups, social and range of aids
with support with employment learning activities & equipment
from mobility & education available in-store
specialists assistance or online

Our support can be life-changing for your patients. They may have access
to NDIS and My Aged Care funding schemes, so please refer to our experts,
who can provide advice and support. You’ll find information on when to
refer to Vision Australia on the following page.

It’s easy to refer to us.


Just visit visionaustralia.org
or call 1300 84 74 66.

Вам также может понравиться